Что происходит с аргументом Паули (который говорит об отсутствии оператора времени) при применении к оператору XXX для некоторых простых систем?

На аргумент Паули обычно ссылаются в литературе, когда утверждается, что в квантовой механике не может быть оператора времени. Этот аргумент можно найти в сноске к стр. 63 книги В. Паули, Общие принципы квантовой механики, с. 63. Springer, Berlin 1980. Соответствующая страница доступна в google books (ищите стр. 63):

https://books.google.co.in/books?id=hVjsCAAAQBAJ&printsec=frontcover&dq=pauli+general+principles+of+quantum+mechanics&hl=en&sa=X&ved=0ahUKEwiOwKX1vvXNAhWJMI8KHSAODC0Q6AEIHDAA#v=onepage&q=discrete%20eigen%20values&f=false

Но я знаю случаи, например частица на окружности или другие случаи с периодическими граничными условиями, когда оператор импульса п имеет только дискретные собственные значения. Но тогда не должен ли аргумент Паули также говорить о том, что нет Икс оператора для таких систем, поскольку коммутационное соотношение между Икс и п совпадает с коммутационным соотношением между Т и Е предполагается Паули?

В этой статье в блоге утверждается, что аргумент недействителен: soulphysics.org/2013/09/…

Ответы (2)

Я не уверен, в чем именно состоит «аргумент Паули», потому что он ссылается на страницы в первом издании « Принципов квантовой механики» Дирака , которые не содержат ничего существенного для моего четвертого издания, но чаще всего говорят, что это ограниченность энергии снизу, запрещающая наивный оператор времени, а не дискретность.

Однако предлагаемый вами сценарий «частица на кольце» не так прост: пространство волновых функций на кольце можно наивно записать как ЧАС "=" { ф е л 2 ( [ 0 , 2 π ] , г ф ) ф ( 0 ) "=" ф ( 2 π ) } , т. е. все возможные квадратично-интегрируемые функции с периодическими граничными условиями. Умножение на «оператор положения» ф , вы должны заметить, что ф не сохраняет граничное условие : ( ф ф ) ( 0 ) "=" ( ф ф ) ( 2 π ) если и только если ф ( 0 ) "=" 0 "=" ф ( 2 π ) . Таким образом, либо у вас есть неестественное ограничение на исчезновение волновых функций в определенной точке, либо вы должны ослабить граничные условия для общей волновой функции (ни одно из них не имеет прямого физического смысла, поскольку значение в одной точке не изменяет плотность вероятности). Подробнее о том, где именно действует коммутационное соотношение, см. в этом моем ответе .

Дело здесь в том, что сами коммутационные соотношения содержат неограниченные операторы и являются довольно патологическими. Если они выполняются только на подмножестве пространства, которое не совпадает с плотной областью, в которой определены операторы, что может произойти, как и в случае частицы на кольце, тогда теорема Стоуна-фон Неймана не выполняется . и можно построить всевозможные контрпримеры.

Формально правильный способ сформулировать теорему Паули таков:

Если гамильтониан ЧАС ограничен снизу или имеет дискретную часть спектра, то нет оператора Т такое, что соотношения Вейля

U ( т ) В ( е ) "=" опыт ( я т е ) В ( е ) U ( т )
для U ( т ) "=" опыт ( я ЧАС т ) , С ( е ) "=" опыт ( я Т е ) держаться везде.

Это следует непосредственно из теоремы Стоуна — фон Неймана, поскольку унитарно эквивалентные операторы обязательно имеют один и тот же спектр, а каноническое представление соотношений Вейля имеет как непрерывные, так и неограниченные операторы. Это означает, что, хотя вы могли бы построить бесконечно малую версию оператора времени в некоторых случаях в некоторой части гильбертова пространства, вы не можете получить из них «ручную» версию CCR, соотношения Вейля. Чтобы увидеть, что отношения Вейля действительно физически необходимы для того, чтобы что-то было правильным «оператором времени», обратите внимание, что U ( т ) В ( е ) U ( т ) "=" опыт ( я т е ) В ( е ) на самом деле просто утверждение, что оператор перевода времени U ( т ) сдвигает значение наблюдаемого времени на т .

Я тоже не мог понять, где соответствующая часть находится в 4-м издании. Первое издание я не смог найти даже в гугл книгах. Но аргумент Паули упоминается в Ballentine, Quantum Mechanics, 1-е издание в разделе 12.3. Но Баллентин упоминает только об ограниченности энергии снизу.
Вы упомянули граничное условие как ф ( 0 ) "=" ф ( 2 π ) . Но я думаю, что граничное условие должно быть ф ( ф ) "=" ф ( ф + 2 π ) для любого ф . Затем ф оператор сохранит граничное условие, только если ф ( ф ) "=" 0 повсюду! Что это будет означать? Или есть какая-то ошибка?
@Curiosa: обратите внимание, что граничное условие ф ( ф ) "=" ф ( ф + 2 π ) не имеет никакого смысла для выбранного мною пространства функций, которые являются функциями на [ 0 , 2 π ] . Однако функции на [ 0 , 2 π ] с ф ( 0 ) "=" ф ( 2 π ) находятся в биекции с функциями на р с ф ( Икс ) "=" ф ( Икс + 2 π ) , так что, по сути, я говорю о такой функции, но презентация как функция на [ 0 , 2 π ] немного облегчает разговор об операторе позиции.
Я не могу заменить S(e) в формуле на V(e), потому что она слишком короткая. Вы могли бы это сделать? банки

Соотношения коммутации между операторами положения и импульса в квантовой механике действительны только тогда, когда действия любого из двух операторов содержатся в области определения оставшегося. В частности, у нас есть:

[ Икс , п ] ψ "=" ( Икс п ) ψ ( п Икс ) ψ .
Для того, чтобы определить вышеизложенное ψ должны находиться в области определения обоих операторов, а также Икс ψ должен находиться в домене п (и наоборот). Для частицы на окружности это как раз не так.

Кроме того, правила коммутации между временем и энергией не являются фактическими правилами коммутации, а скорее ориентировочными утверждениями, которые справедливы только в конкретных случаях.

Я не совсем уверен, что вы подразумеваете под «для частицы на круге это просто не так». Область коммутатора я ф и ф на л 2 ( [ 0 , 2 π ] , г ф ) дан кем-то { ф е л 2 ф е л 2 ф ( 0 ) "=" 0 "=" ф ( 2 π ) } . Но домен коммутатора меньше, чем все пространство, не является чем-то необычным, я не уверен, что это причина того, что аргумент терпит неудачу.
@ACuriousMind Мой аргумент в том, что я не совсем уверен, что коммутатор вообще определен: п ψ в области Икс и наоборот? (Я помню, что один из двух не соответствует граничным требованиям, и это означает, что принцип неопределенности больше не соответствует действительности).
В этом ответе я обсуждаю области задействованных операторов и «лучшую» версию принципа неопределенности . Я подозреваю, что аргумент Паули на самом деле состоит в том, что энергия ограничена снизу . Можно показать, что на полуоси нет согласованных операторов импульса, IIRC - но один оператор ограниченный и непрерывный, а другой неограниченный снизу и сверху и дискретный, как для частицы на интервале/окружности, не проблематичен. Но энергия никогда не бывает неограниченной снизу и непрерывной, так что мы не в этом случае.
О да, именно этот ответ я и имел в виду!